Argument Structure Questions - - Question 17
Scientists, puzzled about the development of penicillin-resistant bacteria in patients who had not been taking penici...
Replies
Naz July 21, 2014
The first sentence of the stimulus states: "Scientists, puzzled about the development of penicillin-resistant bacteria in patients who had not been taking penicillin, believe they have found an explanation." The unstated assumption here is that there were some patients who were taking penicillin who developed penicillin-resistant bacteria.We are asked to find the answer that correctly describes the role of the unstated assumption.
Okay, let's think about the unstated assumption. It's not difficult to see why its possible for patients taking penicillin to develop penicillin-resistant bacteria; as one continues to take penicillin, the probability that the bacteria in their body develops a resistance to it is heightened. However, it is more difficult to explain why people who were NOT taking penicillin would also develop these resistant bacteria.
It makes sense that those who are taking penicillin might develop resistant bacteria, but the same logic does not hold for those who are NOT taking penicillin and are still developing the resistant strain. Thus, taking the fact that patients who had NOT been taking penicillin developed penicillin-resistant bacteria TOGETHER WITH the unstated assumption that some who were taking penicillin also developed this penicillin-resistant bacteria creates our dilemma.
Answer choice (C) states: "It is a point that, in conjunction with the fact that some patients who do not take penicillin develop penicillin-resistant bacteria, generates the problem that prompted the research described in the passage."
This is exactly what the unstated assumption does. Taking the unstated assumption in conjunction with the fact that patients who had not been taking penicillin developed penicillin-resistant bacteria creates the puzzle that caused the scientists to conduct their research and, eventually, find their mercury-related explanation, which was stated in the stimulus.
Hope that was helpful! Please let us know if you have any more questions.
JayDee8732 April 18, 2017
Why is the answer not "A"?
Mehran April 27, 2017
@JayDee8732 (A) is incorrect because it does not pass the "Must Be True" prong of an Argument Structure question.Remember, on Argument Structure questions, there are two requirements: (1) it must be true and (2) it must properly identify the role played.
The unstated assumption that some patients who take penicillin develop bacteria with an immunity to penicillin is not a hypothesis.
It is a statement of fact that when combined with the fact that some patients who do not take penicillin also develop this penicillin-resistant bacteria generates the paradox that the scientists are attempting to explain.
Hope this helps! Please let us know if you have any other questions.
Alec-Julián June 25, 2018
Why is it not D?
Mehran June 27, 2018
The unstated assumption that some patients who take penicillin develop bacteria with an immunity to penicillin is not "the tentative conclusion of previous research." Nothing of that sort is discussed in the stimulus here. Rather, as explained above, this unstated premise is a statement of fact that, when combined with the fact that some patients who *do not* take penicillin *also* develop this penicillin-resistant bacteria, generates the paradox that the scientists are trying to explain. Hope this helps! Please let us know if you have any additional questions.